총 될 때까지 6 면체 주사위를 굴 립니다. 을 초과 하는 평균 금액 은?


11

질문은 다음과 같습니다.

주사위 롤의 합이 M보다 크거나 같을 때까지 공정한 6면 주사위를 반복해서 굴립니다. M = 300 일 때 합계에서 M을 뺀 평균의 평균 및 표준 편차는 얼마입니까?

이런 종류의 질문에 대답하는 코드를 작성해야합니까?

그것에 대한 힌트를주세요. 감사!


1
[self-study]태그를 추가 하고 위키를 읽으십시오 . 그런 다음 지금까지 이해 한 내용, 시도한 내용 및 걸린 위치를 알려주십시오. 문제를 해결하는 데 도움이되는 힌트를 제공합니다.
gung-복직 모니카

2
M = 301 또는 M = 999 가 거의 정확히 같은 결과를 낼 것이라고 믿기 때문에 M=300 을 "매우 큰 M "으로 읽을 수 있다고 생각합니다 . 내가 할 일은 합계에서 M을 뺀 분포를 찾는 것 입니다. M=301M=999M
Henry

답변:


13

확실히 코드를 사용할 수는 있지만 시뮬레이션하지는 않습니다.

나는 "빼기 M"부분을 무시할 것입니다 (끝 부분에서 쉽게 할 수 있습니다).

확률을 재귀 적으로 매우 쉽게 계산할 수 있지만 간단한 추론을 통해 실제 답변 (정확도에 대한 정확도)을 계산할 수 있습니다.

롤을 . 하자 .S t = t i = 1 X iX1,X2,...St=i=1tXi

하자 가장 작은 인덱스 될 .S τMτSτM

P(Sτ=M)=P(got to M6 at τ1 and rolled a 6)+P(got to M5 at τ1 and rolled a 5)++P(got to M1 at τ1 and rolled a 1)=16j=16P(Sτ1=Mj)

여기에 이미지 설명을 입력하십시오

비슷하게

P(Sτ=M+1)=16j=15P(Sτ1=Mj)

P(Sτ=M+2)=16j=14P(Sτ1=Mj)

P(Sτ=M+3)=16j=13P(Sτ1=Mj)

P(Sτ=M+4)=16j=12P(Sτ1=Mj)

P(Sτ=M+5)=16P(Sτ1=M1)

위의 첫 번째와 유사한 방정식은 초기 조건과 원하는 확률 사이의 대수 관계를 얻기 위해 초기 조건 중 하나를 칠 때까지 (적어도 원칙적으로) 되돌릴 수 있습니다 (지루하고 깨달음은 아닙니다) 또는 해당 순방향 방정식을 구성하고 초기 조건에서 순방향으로 실행할 수 있습니다. 이는 수치 적으로 수행하기 쉽고 (내 대답을 확인한 방법입니다). 그러나 우리는 모든 것을 피할 수 있습니다.

포인트의 확률은 이전 확률의 가중 평균을 실행합니다. 이것들은 (초기 적으로 신속하게) 초기 분포로부터의 확률 변동을 완화시킵니다 (문제의 경우 점 0에서의 모든 확률). 그만큼

근사치 (매우 정확한 것)로, 우리는 에서 이 시간 (실제로 가깝습니다) 에서 거의 똑같이 가능해야한다고 말할 수 있습니다. 위에서 우리는 확률을 기록 할 수 있습니다 간단한 비율로 가까워 질 것이며, 정규화되어야하므로 확률을 적어 둘 수 있습니다.M 1 τ 1M6M1τ1

말을하는 것입니다 어느, 우리는 시작의 확률 경우 그 볼 수 있습니다 에 정확히 동일하고, 거기에 점점 6 개 똑같이 가능성 방법이다 을 얻는 5 등을 아래에 에 도착하는 1 가지 방법 .M 1 M M + 1 M + 5M6M1MM+1M+5

즉, 확률은 6 : 5 : 4 : 3 : 2 : 1의 비율이며 1에 합산되므로 적어 두는 것이 쉽지 않습니다.

(나는 R에 그것을했다) 0에서 앞으로 확률 재귀를 실행하여 (오류 오프 누적 수치 라운드까지) 그것을 정확히 계산하는 것은 순서에 차이가 있습니다 .Machine$double.eps( , 말을하는 것입니다 위의 근사에서 내 컴퓨터에을 () 위의 행을 따라 간단한 추론은 정확한 답을 제공 합니다. 왜냐하면 정확한 답이 있어야 할 것으로 예상되는 것처럼 재귀에서 계산 된 답에 가깝기 때문입니다.2.22e-16

여기 내 코드가 있습니다 (대부분 변수를 초기화하는 중입니다. 작업은 모두 한 줄에 있습니다). 코드는 첫 번째 롤 이후에 시작됩니다 (셀에서 0을 넣는 것을 막기 위해 R에서 처리 해야하는 작은 성가심). 각 단계에서 차지할 수있는 가장 낮은 셀을 차지하고 다이 롤에 의해 앞으로 이동합니다 (다음 6 개 셀에 해당 셀의 확률을 확산).

 p = array(data = 0, dim = 305)
 d6 = rep(1/6,6)
 i6 = 1:6
 p[i6] = d6
 for (i in 1:299) p[i+i6] = p[i+i6] + p[i]*d6

(우리는 rollapply(에서 zoo)를 사용 하여이 작업을보다 효율적으로 수행하거나 다른 여러 기능을 수행 할 수는 있지만 명시 적으로 유지하면 번역하기가 더 쉬울 것입니다)

d6마지막 라인의 루프 내부 코드는 이전 값들의 가중 된 평균을 실행하는 구성되도록 1 이상 6의 이산 확률 함수이다. 이 관계는 확률을 매끄럽게합니다 (최근에 관심있는 몇 가지 값까지).

여기에 첫 번째 50 홀수 값 (원으로 표시된 첫 25 개 값)이 있습니다. 각 에서 y 축의 값은 다음 6 개의 셀로 롤 포워드하기 전에 최후 방 셀에 축적 된 확률을 나타냅니다.t

여기에 이미지 설명을 입력하십시오

보시다시피 ( , 각 다이 롤이 걸리는 단계의 평균의 역수)는 매우 빠르게 매끄럽게 유지되고 일정하게 유지됩니다.1/μ

그리고 우리가 에 도달하면 , 그 확률은 사라집니다 (우리는 이상의 값에 대한 확률을 차례로 제시 하지 않기 때문에 )MMM

여기에 이미지 설명을 입력하십시오

따라서 초기 조건의 변동이 완화되기 때문에 에서 까지 의 값 이 동일해야한다는 생각은 분명합니다.M - 6M1M6

추론은 아무것도에 의존하지 않고 이 초기 조건이 씻겨 질 정도로 충분히 커서 에서 이 시간에 거의 똑같이 가능 하므로 분포는 본질적으로 동일합니다. Henry가 의견에서 제안한 것처럼 큰M - 1 M - 6 τ - 1 MMM1M6τ1M

돌이켜 보면, 뺄셈 M을 다루는 Henry의 힌트 (또한 당신의 질문에 있습니다)는 약간의 노력을 아끼지 만 인수는 매우 비슷한 줄을 따릅니다. 당신은 수 있도록함으로써 진행할 수 과 관련된 유사한 방정식 작성 등등 위의 값을합니다.R 0Rt=StMR0

확률 분포에서 확률의 평균과 분산은 간단합니다.

편집 : 최종 위치의 점근 평균과 표준 편차에 뺀 것을 가정합니다 .M

점근 평균 초과는 이고 표준 편차는 입니다. 에서 당신이 걱정하는 가능성이있어보다이 훨씬 더 큰 정도에 정확합니다. 253 M=300253M=300


+1 나는 내 자신을 개발할 때까지이 답변을 완전히 이해하지 못했습니다. 어쩌면 일부 독자는 일러스트레이션 및 시뮬레이션 결과에서 가치를 볼 수 있으므로 답을 열어 두겠습니다.
whuber

1
@ whuber 내 대답은 숙제라고 가정하여 운영했기 때문에 내가 원했던 것보다 훨씬 덜 구체적입니다 (따라서 파생을 너무 많이하거나 코드를 제공하지 않았습니다-더 개요로 의도되었습니다). 나는이 문제에 대한 답을 명확하게 작성하기가 어렵다는 것을 알았습니다 (구체성이 평소보다 더 도움이되는 곳입니다). 실제 숫자와 코드가 포함 된 답변을 제공 했으므로 (답변하지 말아야한다고 생각합니다) 희망적으로 내 답변을 이해하기 쉽도록 할 수있는 일을 할 수 있다고 생각합니다 (보다 명확하고 내 코드를 제공하십시오) .
Glen_b-복지 주 모니카

나는 몇 년 전 어딘가에 이런 종류의 문제에 대해 훨씬 더 나은 설명을 썼습니다. 그것이 어떻게되었는지 기억 / 그림을 볼 수 있다면 여기에 그 중 일부를 포함 시키려고 노력할 것입니다.
Glen_b-복지 주 모니카

@Glen_b는 방정식을 약간 이해했습니다. 나는 초보자입니다. 이런 생각을 시작하는 방법? 이런 종류의 목적으로 추천 할만한 책이 있습니까? 답장이 크게 도움이 될 것입니다.
일반적인 용의자

평범한 용의자-나는 긴 보드처럼 게임 보드를 상상하고 "문제의 조건에 맞는 방식으로이 공간에 어떻게 접근 할 수 있는가?" 나는 "M"이라고 표시된 공간을 위해, 그 다음의 공간을 위해 그렇게했습니다. 나는 시작 셀 근처에있는 것을 상상하고 "내가 여기 있다면, 다음 기회는 무엇일까?" 방정식은 모두 그 질문에 대한 답일뿐입니다.
Glen_b-복지 주 모니카

8

하자 (각 시퀀스에서 시작으로 주사위 롤 부분합 시퀀스들의 집합 ). 임의의 정수 , 을 이 시퀀스에 나타나는 이벤트라고 하자 . 그건,0 n E n nΩ0nEnn

En={ωΩ|nω}.

를 에서 과 같거나 초과 하는 첫 번째 값으로 정의하십시오 . 질문은 속성을 묻습니다 . 의 정확한 분포를 얻을 수 있으며 모든 것이 뒤 따릅니다.ω M X M - MXM(ω)ωMXMMXM

먼저 입니다. 이벤트 분할함으로써 있는 직전의 값에 따라 및시키는 얼굴 관측 확률 수 상기 다이의 하나의 롤 ( )에 따라XM(ω)M{0,1,2,3,4,5}XMM=kωp(i)=1/6ii=1,2,3,4,5,6

Pr(XMM=k)=j=k6Pr(EM+kj)p(j)=16j=k6Pr(EM+kj).

이 시점에서 우리는 가장 작은 제외하고는 아주 좋은 근사치 인 이라고 heuristically 주장 할 수있었습니다롤의 예상 값이 이고 그 역수는 특정 값에 대한 제한적이고 안정적인 장기 빈도 여야하기 때문 입니다.M

Pr(Ei)2/7.
(1+2+3+4+5+6)/6=7/2ω

이를 입증하는 엄격한 방법은 가 발생할 수있는 방법을 고려합니다 . 어느 발생 이후의 롤이 있었다 ; 또는 가 발생하고 후속 롤은 . 또는 ... 또는 발생하고 후속 롤은 입니다. 이것은 가능성의 철저한 분할입니다.EiEi11Ei22Ei66

Pr(Ei)=j=16Pr(Eij)p(j)=16j=16Pr(Eij).

이 순서의 초기 값은

Pr(E0)=1;Pr(Ei)=0,i=1,2,3,.

그림 : E_i 플롯

에 대한 이 플롯은 수평 점선으로 표시되는 확률이 상수 로 얼마나 빠르게 안정되는지 보여줍니다 .Pr(Ei)i2/7

이러한 재귀 시퀀스에 대한 표준 이론이 있습니다. 함수, Markov 체인 또는 대수 조작을 생성하여 개발할 수 있습니다. 일반적인 결과는 대한 닫힌 형식의 수식 이 존재한다는 것입니다. Pr(Ei)다항식의 근의 상수와 제곱의 선형 조합입니다.ith

x6p(1)x5p(2)x4p(3)x3p(6)=x6(x5+x4+x3+x2+x+1)/6.

이 루트의 가장 큰 크기는 대략 입니다. 배정 밀도 부동 소수점 표현에서 는 기본적으로 0입니다. 그러므로, , 우리는 완전히 제외한 모든 정수를 무시할 수있다. 이 상수는 입니다.exp(0.314368)exp(36.05)i36.05/0.314368=1152/7

결과적으로 인 경우 모든 실제적인 목적으로 취할 수 있습니다 .M=300115EM+kj=2/7

Pr(XMM=(0,1,2,3,4,5))=(27)(16)(6,5,4,3,2,1).

이 분포의 평균과 분산을 계산하는 것은 간단하고 쉽습니다.


다음은 R이러한 결론을 확인하기 위한 시뮬레이션입니다. 통해 거의 100,000 개의 시퀀스를 생성 하고 의 값을 표로 만들고 테스트를 적용 하여 결과가 전술 한 내용과 일치하는지 여부를 평가합니다. 의 p- 값 (이 경우) 은 일관성을 나타내기에 충분히 큽니다.X 300300 χ 2 0.1367M+5=305X300300χ20.1367

M <- 300
n.iter <- 1e5
set.seed(17)
n <- ceiling((2/7) * (M + 3*sqrt(M)))
dice <- matrix(ceiling(6*runif(n*n.iter)), n, n.iter)
omega <- apply(dice, 2, cumsum)
omega <- omega[, apply(omega, 2, max) >= M+5]
omega[omega < M] <- NA
x <- apply(omega, 2, min, na.rm=TRUE)
count <- tabulate(x)[0:5+M]
(cbind(count, expected=round((2/7) * (6:1)/6 * length(x), 1)))
chisq.test(count, p=(2/7) * (6:1)/6)
당사 사이트를 사용함과 동시에 당사의 쿠키 정책개인정보 보호정책을 읽고 이해하였음을 인정하는 것으로 간주합니다.
Licensed under cc by-sa 3.0 with attribution required.